Difference between revisions of "1999 AIME Problems/Problem 14"

(Solution)
(solutions by (1) joml88 (2) 1234567890)
Line 1: Line 1:
 
== Problem ==
 
== Problem ==
Point <math>\displaystyle P_{}</math> is located inside traingle <math>\displaystyle ABC</math> so that angles <math>\displaystyle PAB, PBC,</math> and <math>\displaystyle PCA</math> are all congruent.  The sides of the triangle have lengths <math>\displaystyle AB=13, BC=14,</math> and <math>\displaystyle CA=15,</math> and the tangent of angle <math>\displaystyle PAB</math> is <math>\displaystyle m/n,</math> where <math>\displaystyle m_{}</math> and <math>\displaystyle n_{}</math> are relatively prime positive integers.  Find <math>\displaystyle m+n.</math>
+
Point <math>P_{}</math> is located inside traingle <math>ABC</math> so that angles <math>PAB, PBC,</math> and <math>PCA</math> are all congruent.  The sides of the triangle have lengths <math>AB=13, BC=14,</math> and <math>CA=15,</math> and the tangent of angle <math>PAB</math> is <math>m/n,</math> where <math>m_{}</math> and <math>n_{}</math> are relatively prime positive integers.  Find <math>m+n.</math>
  
 +
__TOC__
 
== Solution ==
 
== Solution ==
{{solution}}
+
[[Image:1999_AIME-14.png]]
 +
=== Solution 1 ===
 +
Drop perpendiculars from <math>P</math> to the three sides of <math>\triangle ABC</math> and let them meet <math>\overline{AB}, \overline{BC},</math> and <math>\overline{CA}</math> at <math>D, E,</math> and <math>F</math> respectively.  Let <math>BE = x, CF = y,</math> and <math>AD = z</math>.  We have that
 +
<cmath>
 +
\begin{eqnarray*} DP & = & z\tan \theta \\
 +
EP & = & x\tan \theta \\
 +
FP & = & y\tan \theta \end{eqnarray*}
 +
</cmath>
 +
We can then use the tool of calculating area in two ways
 +
<cmath>
 +
\begin{eqnarray*} [ABC] & = & [PAB] + [PBC] + [PCA] \\
 +
& = & \frac 12 (13)(z\tan \theta) + \frac 12 (14)(x\tan\theta) + \frac 12 (15)(y\tan\theta) \\
 +
& = & \frac 12 \tan\theta(13z + 14x + 15y) \end{eqnarray*}
 +
</cmath>
 +
On the other hand
 +
<cmath>
 +
\begin{eqnarray*} [ABC] & = & \sqrt {s(s - a)(s - b)(s - c)} \\
 +
& = & \sqrt {21\cdot 6\cdot 7\cdot 8} \\
 +
& = & 84 \end{eqnarray*}
 +
</cmath>
 +
We still need <math>13z + 14x + 15y</math> though.  We have all these right triangles and we haven't even touched Pythagoras.  So we give it a shot
 +
<cmath>
 +
\begin{eqnarray} x^2 + x^2\tan^2\theta & = & z^2\tan^2\theta + (13 - z)^2 \\
 +
z^2 + z^2\tan^2\theta & = & y^2\tan^2\theta + (15 - y)^2 \\
 +
y^2 + y^2\tan^2\theta & = & x^2\tan^2\theta + (14 - x)^2 \\
 +
\end{eqnarray*}
 +
</cmath>
 +
But then <math>(1) + (2) + (3)</math> gives
 +
<cmath>
 +
\begin{eqnarray*} x^2 + y^2 + z^2 & = & (14 - x)^2 + (15 - y)^2 + (13 - z)^2 \\
 +
\Rightarrow 13z + 14x + 15y & = & 295 \end{eqnarray*}
 +
</cmath>
 +
Recall that we found that <math>[ABC] = \frac 12 \tan\theta(13z + 14x + 15y) = 84</math>.  Plugging in <math>13z + 14x + 15y = 295</math> we get <math>\tan \theta = \frac {168}{295}</math> giving us <math>\boxed{463}</math> for an answer.
 +
 
 +
=== Solution 2 ===
 +
Let <math>AB = c, BC = a, AC = b, PA = x, PB = y, PC = z</math>.
 +
 
 +
So by the Law of Cosines, we have:
 +
<math>x^2 = z^2 + b^2 - 2bz\cos{\theta}</math>
 +
<math>y^2 = x^2 + c^2 - 2cx\cos{\theta}</math>
 +
<math>z^2 = y^2 + a^2 - 2ay\cos{\theta}</math>
 +
 
 +
Adding these equations and rearranging, we have:
 +
<math>a^2 + b^2 + c^2 = (2bz + 2cx + 2ay)\cos{\theta}</math>                (1)
 +
 
 +
Now <math>[CAP] + [ABP] + [BCP] = [ABC] = \sqrt {(21)(8)(7)(6)} = 84</math>, by Heron's Formula.
 +
 
 +
Now the area of a triangle, <math>[A] = \frac {mn\sin{\beta}}{2}</math>, where <math>m</math> and <math>n</math> are sides on either side of an angle, <math>\beta</math>. So,
 +
<math>[CAP] = \frac {bz\sin{\theta}}{2}</math>.
 +
<math>[ABP] = \frac {cx\sin{\theta}}{2}</math>.
 +
<math>[BCP] = \frac {ay\sin{\theta}}{2}</math>
 +
 
 +
So adding these equations yields:
 +
<math>[ABC] = 84 = \frac {(bz + cx + ay)\sin{\theta}}{2}</math>
 +
<math>\Rightarrow 168 = (bz + cx + ay)\sin{\theta}</math>                                        (2)
 +
 
 +
Dividing (2) by (1), we have:
 +
<math>\frac {168}{a^2 + b^2 + c^2} = \frac {(bz + cx + ay)\sin{\theta}}{(2bz + 2cx + 2ay)\cos{\theta}}</math>
 +
<math>\Rightarrow \tan{\theta} = \frac {336}{a^2 + b^2 + c^2} = \frac {336}{14^2 + 15^2 + 13^2} = \frac {336}{590} = \frac {168}{295}</math>
 +
 
 +
So <math>m + n = 168 + 295 = \boxed{463}</math>.
  
 
== See also ==
 
== See also ==
 +
*[[Brocard point]]
 
{{AIME box|year=1999|num-b=13|num-a=15}}
 
{{AIME box|year=1999|num-b=13|num-a=15}}
 +
 +
[[Category:Intermediate Geometry Problems]]

Revision as of 21:46, 18 October 2007

Problem

Point $P_{}$ is located inside traingle $ABC$ so that angles $PAB, PBC,$ and $PCA$ are all congruent. The sides of the triangle have lengths $AB=13, BC=14,$ and $CA=15,$ and the tangent of angle $PAB$ is $m/n,$ where $m_{}$ and $n_{}$ are relatively prime positive integers. Find $m+n.$

Solution

File:1999 AIME-14.png

Solution 1

Drop perpendiculars from $P$ to the three sides of $\triangle ABC$ and let them meet $\overline{AB}, \overline{BC},$ and $\overline{CA}$ at $D, E,$ and $F$ respectively. Let $BE = x, CF = y,$ and $AD = z$. We have that \begin{eqnarray*} DP & = & z\tan \theta \\ EP & = & x\tan \theta \\ FP & = & y\tan \theta \end{eqnarray*} We can then use the tool of calculating area in two ways \begin{eqnarray*} [ABC] & = & [PAB] + [PBC] + [PCA] \\ & = & \frac 12 (13)(z\tan \theta) + \frac 12 (14)(x\tan\theta) + \frac 12 (15)(y\tan\theta) \\ & = & \frac 12 \tan\theta(13z + 14x + 15y) \end{eqnarray*} On the other hand \begin{eqnarray*} [ABC] & = & \sqrt {s(s - a)(s - b)(s - c)} \\ & = & \sqrt {21\cdot 6\cdot 7\cdot 8} \\ & = & 84 \end{eqnarray*} We still need $13z + 14x + 15y$ though. We have all these right triangles and we haven't even touched Pythagoras. So we give it a shot

\begin{eqnarray} x^2 + x^2\tan^2\theta & = & z^2\tan^2\theta + (13 - z)^2 \\
z^2 + z^2\tan^2\theta & = & y^2\tan^2\theta + (15 - y)^2 \\
y^2 + y^2\tan^2\theta & = & x^2\tan^2\theta + (14 - x)^2 \\
\end{eqnarray*} (Error compiling LaTeX. Unknown error_msg)

But then $(1) + (2) + (3)$ gives \begin{eqnarray*} x^2 + y^2 + z^2 & = & (14 - x)^2 + (15 - y)^2 + (13 - z)^2 \\ \Rightarrow 13z + 14x + 15y & = & 295 \end{eqnarray*} Recall that we found that $[ABC] = \frac 12 \tan\theta(13z + 14x + 15y) = 84$. Plugging in $13z + 14x + 15y = 295$ we get $\tan \theta = \frac {168}{295}$ giving us $\boxed{463}$ for an answer.

Solution 2

Let $AB = c, BC = a, AC = b, PA = x, PB = y, PC = z$.

So by the Law of Cosines, we have: $x^2 = z^2 + b^2 - 2bz\cos{\theta}$ $y^2 = x^2 + c^2 - 2cx\cos{\theta}$ $z^2 = y^2 + a^2 - 2ay\cos{\theta}$

Adding these equations and rearranging, we have: $a^2 + b^2 + c^2 = (2bz + 2cx + 2ay)\cos{\theta}$ (1)

Now $[CAP] + [ABP] + [BCP] = [ABC] = \sqrt {(21)(8)(7)(6)} = 84$, by Heron's Formula.

Now the area of a triangle, $[A] = \frac {mn\sin{\beta}}{2}$, where $m$ and $n$ are sides on either side of an angle, $\beta$. So, $[CAP] = \frac {bz\sin{\theta}}{2}$. $[ABP] = \frac {cx\sin{\theta}}{2}$. $[BCP] = \frac {ay\sin{\theta}}{2}$

So adding these equations yields: $[ABC] = 84 = \frac {(bz + cx + ay)\sin{\theta}}{2}$ $\Rightarrow 168 = (bz + cx + ay)\sin{\theta}$ (2)

Dividing (2) by (1), we have: $\frac {168}{a^2 + b^2 + c^2} = \frac {(bz + cx + ay)\sin{\theta}}{(2bz + 2cx + 2ay)\cos{\theta}}$ $\Rightarrow \tan{\theta} = \frac {336}{a^2 + b^2 + c^2} = \frac {336}{14^2 + 15^2 + 13^2} = \frac {336}{590} = \frac {168}{295}$

So $m + n = 168 + 295 = \boxed{463}$.

See also

1999 AIME (ProblemsAnswer KeyResources)
Preceded by
Problem 13
Followed by
Problem 15
1 2 3 4 5 6 7 8 9 10 11 12 13 14 15
All AIME Problems and Solutions